Đến nội dung

EVEREST! nội dung

Có 63 mục bởi EVEREST! (Tìm giới hạn từ 30-03-2020)



Sắp theo                Sắp xếp  

#145935 Bài mới

Đã gửi bởi EVEREST! on 04-02-2007 - 16:28 trong Bất đẳng thức và cực trị

Tôi có 2 cách như sau:
C1)
bdt$ \Leftrightarrow (a^2x+b^2y+c^2z)^2 \geq 16S^2(xy+yz+zx)$
$\Leftrightarrow (a^2x+b^2y+c^2z)^2 \geq(xy+yz+zx)(2a^2b^2+2b^2c^2+2c^2a^2-a^4-b^4-c^4)$
Đến đây có thể khai triển và dùng AM-GM.
C2)
Đặt cotgP=x,cotgQ=y,cotgR=z với P,Q,R là 3 góc 1 tam giác.
Đặt $ S_{ABC} =S, S_{PQR}=S'.$
bdt cần c/m $ \Leftrightarrow a^2cotgP+b^2cotgQ+c^2cotgR \geq 4S.$

C/m tiếp cách 2 như sau:goi p,q,r là các cạnh tam giác PQR
bdt$ \Leftrightarrow a^2(q^2+r^2-p^2)+b^2(p^2+r^2-q^2)+c^2(p^2+q^2-r^2) \geq 16SS'$(sử dụng ct:$cotgP= \dfrac{q^2+r^2-p^2}{4S'}$)
$ \Leftrightarrow a^2(2q^2-2pqcosR)+b^2(2p^2-2pqcosR)+2c^2xycosR \geq 4absinCpqsinR $
$ \Leftrightarrow 2(aq-bp)^2+4abpq(1-cos(C-R) \geq 0 $ (đúng)
Dấu = xảy ra khi 2 tam giác đồng dạng



#145555 1 bài về mạng lưới ô vuông!

Đã gửi bởi EVEREST! on 02-02-2007 - 16:19 trong Tổ hợp và rời rạc

Cho hình H có diện tích lớn hơn 1.CM: tồn tại 2 điểm A,B thuộc H t/m:$AB^2$là số nguyên dương.



#143172 Bài mới

Đã gửi bởi EVEREST! on 19-01-2007 - 17:03 trong Bất đẳng thức và cực trị

Tôi có 2 cách như sau:
C1)
bdt$ \Leftrightarrow (a^2x+b^2y+c^2z)^2 \geq 16S^2(xy+yz+zx)$
$\Leftrightarrow (a^2x+b^2y+c^2z)^2 \geq(xy+yz+zx)(2a^2b^2+2b^2c^2+2c^2a^2-a^4-b^4-c^4)$
Đến đây có thể khai triển và dùng AM-GM.
C2)
Đặt cotgP=x,cotgQ=y,cotgR=z với P,Q,R là 3 góc 1 tam giác.
Đặt $ S_{ABC} =S, S_{PQR}=S'.$
bdt cần c/m $ \Leftrightarrow a^2cotgP+b^2cotgQ+c^2cotgR \geq 4S.$



#143148 Dễ thôi!

Đã gửi bởi EVEREST! on 19-01-2007 - 16:18 trong Công thức lượng giác, hàm số lượng giác

Bài này có thể làm như sau:
Biến đổi:$ OD^2=R^2-(p-b)(p-c) \geq R^2- \dfrac{a^2}{4}$
$ \Rightarrow OD^2+OE^2+OF^2$
$ \geq 3R^2- \dfrac{a^2+b^2+c^2}{4} $
$ \geq 3R^2- \dfrac{8R^2+4r^2}{4} =R^2-r^2$



#142493 Đặt xe!

Đã gửi bởi EVEREST! on 16-01-2007 - 17:15 trong Tổ hợp và rời rạc

Đặt n con xe lên bàn cờ n.n thỏa mãn chúng không ăn nhau và không thuộc hai đường chéo chính.(n>2)
Hỏi có bao nhiêu cách .



#142487 Tính các góc cuả tam giác ABC

Đã gửi bởi EVEREST! on 16-01-2007 - 17:02 trong Công thức lượng giác, hàm số lượng giác

Tính các góc cuả tam giác ABC ; biết $\Large -4sin^3A + sin2B + sin2C + 4sinA = 2\sqrt{2} $

Bài này dễ thôi!
Vế trái=$-4sin^3A+4sinA+2sinAcos(B-C)$
$ \leq -4sin^3A+6sinA=2sinA(3-2sin^2A)= \sqrt{4sin^2 A(3-2sin^2A)(3-2sin^2A)} \leq 2 \sqrt{2}$(Sử dụng AM-GM)
Dấu= xảy ra khi tam giác ABC đều.



#142483 Dễ thôi!

Đã gửi bởi EVEREST! on 16-01-2007 - 16:54 trong Công thức lượng giác, hàm số lượng giác

O là chi đó bạn??

O là tâm đường tròn ngoại tiếp tam giác!



#142065 Dễ thôi!

Đã gửi bởi EVEREST! on 14-01-2007 - 15:41 trong Công thức lượng giác, hàm số lượng giác

Cho tam giác ABC nhọn, đường tròn nội tiếp tiếp xúc với các cạnh tạiD,E,F.CMR:
$OD^2+OE^2+OF^2 \geq R^2-r^2$



#142061 bài hình khá dễ

Đã gửi bởi EVEREST! on 14-01-2007 - 15:15 trong Hình học phẳng

Cho tam giác ABC,1 đt qua trọng tâm G cắt 2 cạnh AB,AC tại E,F.CMR:
$\dfrac{EA}{EB} \dfrac{FA}{FC} \geq 4$



#142056 bài hình khá dễ

Đã gửi bởi EVEREST! on 14-01-2007 - 14:55 trong Hình học phẳng

cho tứ giác ABCD
$DA \cap CB =K,AB \cap DC =L,AC \cap KL =G,DB \cap KL= F$
cmr:$\dfrac {KF}{FL}=\dfrac{KG}{GL}$

Bài này chỉ cần sử dụng Menenauyt cho các tam giác:
Tam giác KLC(B,E,D)
Tam giác KLD(G,C,A)
Tam giác KCD(A,L,B)



#141506 BDTBDTBDT!

Đã gửi bởi EVEREST! on 11-01-2007 - 16:30 trong Bất đẳng thức và cực trị

Cho a,b,c>0 t/m:ab+bc+ca=1.CMR:
$4(a+b+c)- \dfrac{1}{abc} \leq \sqrt{3} $

Bài này còn có thể dùng lượng giác hóa!



#141503 BDT!

Đã gửi bởi EVEREST! on 11-01-2007 - 16:26 trong Bất đẳng thức và cực trị

Cho$ a,b,c \geq 0$ t/m:a+b+c=1.CMR:
$2(a^2+b^2+c^2)+9abc \geq 1.$

Thực ra bài này dễ thôi:
cũng đưa bdt về:$1+9abc \geq 2(ab+bc+ca)$
$ \Leftrightarrow (a+b-c)(b+c-a)(c+a-b) \leq abc$(cái này thì đơn giản rùi)



#141092 BDT!

Đã gửi bởi EVEREST! on 09-01-2007 - 17:00 trong Bất đẳng thức và cực trị

Cho$ a,b,c \geq 0$ t/m:a+b+c=1.CMR:
$2(a^2+b^2+c^2)+9abc \geq 1.$



#141071 Một bài cho vui

Đã gửi bởi EVEREST! on 09-01-2007 - 16:31 trong Công thức lượng giác, hàm số lượng giác

Tìm đk của tam giác ABC để $ \dfrac{(a+b)(b+c)(c+a)}{4abc}=\dfrac{R}{r}$

Bài này sử dụng các công thức:
$ \dfrac{r}{R} =4sin( \dfrac{A}{2}) sin( \dfrac{B}{2})sin( \dfrac{C}{2})$ .
và a=2RsinA,b=2RsinB,c=2RsinC đưa bài toán về:
$cos( \dfrac{A-B}{2})cos( \dfrac{B-C}{2})cos( \dfrac{C-A}{2})=1$
$ \Rightarrow$ tam giác ABC đều.



#140460 BDTBDTBDT!

Đã gửi bởi EVEREST! on 06-01-2007 - 16:45 trong Bất đẳng thức và cực trị

Cho a,b,c>0 t/m:ab+bc+ca=1.CMR:
$4(a+b+c)- \dfrac{1}{abc} \leq \sqrt{3} $



#140457 Khó đây!

Đã gửi bởi EVEREST! on 06-01-2007 - 16:40 trong Tổ hợp và rời rạc

Cho S={1,2,..,n^2+n+1}.Giả sử F là họ các tập con của tập S và t/m:
i)Mỗi tập của họ F chứa nhiều hơn n^2 phần tử.
ii)Mỗi phần tử của tập S xuất hiện ở nhiều hơn n^2 tập của họ F.
CMR:Tồn tại 2 tập của họ F khi hợp lại đúng bằng tập S.



#140451 bat dang thuc de

Đã gửi bởi EVEREST! on 06-01-2007 - 16:19 trong Công thức lượng giác, hàm số lượng giác

Hình như lovewin ghi nhầm đề.Đề bài phải như thế này:
$ \sqrt{3} +cotgA+cotgB+cotgC \leq \dfrac{1}{sinA} + \dfrac{1}{sinB} + \dfrac{1}{sinC} $
Để c/m bdt này ta làm như sau:
bdt$ \Leftrightarrow \sqrt{3} \leq \sum \dfrac{1-cosA}{sinA}$
$ \Leftrightarrow \sqrt{3} \leq \sum tg \dfrac{A}{2}$
Cái này dùng Jensen là ra!
(Để viết đúng công thức bạn đặt xem phần cách ghõ công thức)



#140107 Cực khó!

Đã gửi bởi EVEREST! on 04-01-2007 - 16:08 trong Công thức lượng giác, hàm số lượng giác

Đề đóm chán quá

Đề bài có vấn đề gì đâu hả bạn TUNGTUNG!



#139232 vui vui

Đã gửi bởi EVEREST! on 22-12-2006 - 17:09 trong Quán hài hước

Hay đấy!



#139224 Cực khó!

Đã gửi bởi EVEREST! on 22-12-2006 - 16:49 trong Công thức lượng giác, hàm số lượng giác

CMR: với mọi tam giác ABC nội tiếp trong đường tròn bán kính đơn vị điều kiện cần và đủ để tam giác ABC đều là:




#139218 Vấn đề nhỏ ( Usa 1980)

Đã gửi bởi EVEREST! on 22-12-2006 - 16:25 trong Bất đẳng thức và cực trị

Bài này có thể tổng quát.C/m tương tự như bài trên!



#139035 TẬP HỢP!

Đã gửi bởi EVEREST! on 21-12-2006 - 17:09 trong Tổ hợp và rời rạc

Tập X gồm n phần tử .Với 2 tập con bất kì A,B( có thể A=B) ta tính số phần tử của .CMR: Tổng của tất cả các số nhận được là



#139028 Bài này hay đây!

Đã gửi bởi EVEREST! on 21-12-2006 - 17:04 trong Tổ hợp và rời rạc

Đ/S: a)
b)



#139023 Khá hay!

Đã gửi bởi EVEREST! on 21-12-2006 - 16:59 trong Tổ hợp và rời rạc

Với mỗi tập hợpX ta thành lập 1 tập X'={a-b/}.Cho S={1,2,..,2000}.A,B là 2 tập con của X sao cho .CMR:



#139014 Bài này hay đây!

Đã gửi bởi EVEREST! on 21-12-2006 - 16:44 trong Tổ hợp và rời rạc

Cho các số,.., :cap t/m:

Xét tập T={}(2 phần tử bằng nhau chỉ tính làm 1)
i)Cho :cap =
Tìm số phần tử của T .(dễ)
ii)Cho :cap =
Tìm số phần tử của T.(khó)